LSAT and Law School Admissions Forum

Get expert LSAT preparation and law school admissions advice from PowerScore Test Preparation.

 Administrator
PowerScore Staff
  • PowerScore Staff
  • Posts: 8917
  • Joined: Feb 02, 2011
|
#23731
Complete Question Explanation

Weaken. The correct answer choice is (B)

The argument is that since the tax reform laws will reduce the tax burden on low-income taxpayers by $100 to $300, the reform is clearly in the interest of low- income taxpayers.

The argument is not at all convincing, because it focuses on a possibly minor alteration in the taxation level without any concentration on other effects of the tax reform. For example, the tax reform could reduce benefits or programs that low-income taxpayers receive.

Since you are asked to weaken the conclusion, you should attack the flaw, which is that the argument is based on glaringly incomplete information.

Answer choice (A): If the new tax codes help people save money because they will not have to go to an accountant, that might help taxpayers, even a few low-income taxpayers. This choice does not weaken the conclusion, so this choice is wrong.

Answer choice (B): This is the correct answer choice. If the result of the reform is that rent increases for low-income taxpayers by $40 a month, there is a chance that low-income taxpayers will then incur a $480 expense, which their refund may not fully cover.

Answer choice (C): The voting behavior of a population does not demonstrate what its rational interest is, because a population can be ignorant or stupid. In any case, this choice could strengthen the argument by making the reform seem desirable to low-income taxpayer-voters, so this choice is wrong.

Answer choice (D): If the new reforms allow low-income taxpayers to deduct child-care expenses from their taxes, the new reforms might be even better than expected, so this choice strengthens the conclusion, and is incorrect.

Answer choice (E): If many low-income taxpayers are totally exempted from taxes because of the reforms, the reforms might more likely be in their interests, so this choice somewhat strengthens the conclusion and is incorrect.
 andriana.caban
  • Posts: 142
  • Joined: Jun 23, 2017
|
#63496
Administrator wrote:Complete Question Explanation

Answer choice (D): If the new reforms allow low-income taxpayers to deduct child-care expenses from their taxes, the new reforms might be even better than expected, so this choice strengthens the conclusion, and is incorrect.
Hi, I was deciding between (B) and (D) and reluctantly choose D because I incorrectly assumed that, because the new tax reform laws now ("NTRL") permit middle-income tax payers to deduct child-care expenses from their taxes, the NTRL laws is also in the interest of the middle-class and not just the low-income tax payers. Can you please explain why my logic is incorrect?
 Jay Donnell
PowerScore Staff
  • PowerScore Staff
  • Posts: 144
  • Joined: Jan 09, 2019
|
#63504
Hi Andriana!

The answer in D fails to weaken the argument because the ability for low-income taxpayers to deduct child-care expenses from their taxes provides a benefit rather than a disadvantage. True, the answer does imply that this benefit would also extend to the middle-income taxpayers, but that in itself doesn't bring any negative effects toward those in the lower-income levels.

Had the conclusion claimed that this new policy would benefit only the low-income taxpayers, then D could have been a more effective Weaken answer by showing the benefits fall outside of just that specific demographic. However, as the conclusion merely stated that the new tax reform laws would benefit the low-income group, an additional benefit being shown to effect both low and middle-income taxpayers does not weaken the argument.

Hope that helps!
 andriana.caban
  • Posts: 142
  • Joined: Jun 23, 2017
|
#63530
Wow! I understand completely after your explanation, thank you!

Get the most out of your LSAT Prep Plus subscription.

Analyze and track your performance with our Testing and Analytics Package.